2009 USAMO Problems/Problem 5
Problem
Trapezoid , with
, is inscribed in circle
and point
lies inside triangle
. Rays
and
meet
again at points
and
, respectively. Let the line through
parallel to
intersect
and
at points
and
, respectively. Prove that quadrilateral
is cyclic if and only if
bisects
.
Solution 1
We will use directed angles in this solution. Extend to
as follows:
![[asy] import cse5; import graph; import olympiad; dotfactor = 3; unitsize(1.5inch); path circle = Circle(origin, 1); draw(circle); pair A = (-.6, .8), B = (.6, .8), C = (.9, -sqrt(.19)), D = (-.9, -sqrt(.19)), G = bisectorpoint(C, B, D); draw(A--B--C--D--cycle); draw(B--D); dot("$A$", A, NW); dot("$B$", B, NE); dot("$C$", C, SE); dot("$D$", D, SW); dot("$G$", G, dir(40)); pair P = IP(L(A, G, 10, 10), circle, 1), Q = IP(L(B, G, 10, 10), circle, 1); draw(A--P--C); draw(B--Q); dot("$P$", P, SE); dot("$Q$", Q, S); pair R = IP((-1, G.y)--(1, G.y), B--D), S = IP((-1, G.y)--(1, G.y), B--C); draw(P--Q--R--S--cycle); dot("$R$", R, N); dot("$S$", S, E); pair T = IP(L(Q, R, 10, 10), circle, 1); draw(R--T--C, dashed); draw(T--B, dashed); dot("$T$", T, NW); [/asy]](http://latex.artofproblemsolving.com/1/1/7/117688c890349ac2b07c260b638aa6973c0f0c25.png)
If:
Note that
Thus,
is cyclic.
Also, note that is cyclic because
depending on the configuration.
Next, we have are collinear since
Therefore, so
is cyclic.
Only If: These steps can be reversed.
Solution 2 (Projective)
Extend to
, and let line
intersect
at
and another point
, as shown:
![[asy] import cse5; import graph; import olympiad; dotfactor = 3; unitsize(1.5inch); path circle = Circle(origin, 1); draw(circle); pair A = (-.6, .8), B = (.6, .8), C = (.9, -sqrt(.19)), D = (-.9, -sqrt(.19)), G = bisectorpoint(C, B, D); draw(A--B--C--D--cycle); draw(B--D); dot("$A$", A, NW); dot("$B$", B, NE); dot("$C$", C, SE); dot("$D$", D, SW); dot("$G$", G, dir(40)); pair P = IP(L(A, G, 10, 10), circle, 1), Q = IP(L(B, G, 10, 10), circle, 1); draw(A--P); draw(B--Q); dot("$P$", P, SE); dot("$Q$", Q, S); pair R = IP((-1, G.y)--(1, G.y), B--D), S = IP((-1, G.y)--(1, G.y), B--C); draw(P--Q--R--S); dot("$R$", R, N); dot("$S$", S, E); pair T = IP(L(Q, R, 10, 10), circle, 1); draw(Q--T); pair V = IP(L(P, S, 10, 10), circle, 1); draw(T--V); draw(P--V, dotted); dot("$T$", T, NW); dot("$V$", V, NE); [/asy]](http://latex.artofproblemsolving.com/f/b/9/fb97d4173f22b4e32b34e763023736dea8d8b663.png)
If:
Suppose that , and
. Pascal's theorem on the tuple
implies that the points
,
, and
are collinear. However,
and
are symmetrical with respect to the axis of symmetry of trapezoid
, and
and
are also symmetrical with respect to the axis of symmetry of
(as
is the midpoint of
, and
). Since
,
and
are symmetric with respect to the axis of symmetry of trapezoid
. This implies that line
is equivalent to line
. Thus,
lies on line
. However,
, so this implies that
.
Now note that is cyclic. Since
,
. However,
. Therefore,
is cyclic.
Only If:
Consider the same setup, except is no longer the midpoint of
. Note that
must be parallel to
in order for
to be cyclic. We claim that
and hope to reach a contradiction. Pascal's theorem on the tuple
implies that
,
, and
are collinear. However, there exists a unique point
such that
,
, and
are concurrent. By If,
must be the midpoint of
in order for the concurrency to occur; hence,
. Then
, since
. However, this is a contradiction, so therefore
cannot be parallel to
and
is not cyclic.
Solution by TheBoomBox77
See Also
2009 USAMO (Problems • Resources) | ||
Preceded by Problem 4 |
Followed by Problem 6 | |
1 • 2 • 3 • 4 • 5 • 6 | ||
All USAMO Problems and Solutions |
The problems on this page are copyrighted by the Mathematical Association of America's American Mathematics Competitions.